Help Calculating Pi using Arctangent formula

Click For Summary
The discussion focuses on using the Arctangent formula to calculate pi to 53 significant digits. A user attempted the calculation in MAPLE 13 but received an incorrect result of 3.17 instead of the expected 3.14. Another participant suggests that the issue may stem from how the calculation is structured in MAPLE, recommending the use of brackets around the expression after "evalf." Additionally, it is mentioned that not all terms up to x^{51} may be necessary to achieve the desired accuracy, and the remainder theorem for Taylor expansions could help determine the required number of terms. The conversation emphasizes troubleshooting the calculation method in MAPLE for accurate results.
PolyFX
Messages
30
Reaction score
0

Homework Statement


Using the Arctangent formula
pi = 16 * arctan (1 / 5) - 4*arctan(1 / 239) to calculate the value of pi to 53 significant digits.

Homework Equations



The power series of arctangent(x) is = x − x^3/3 + x^5/5 − x^7/7 + x^9/9...

The Attempt at a Solution



http://tinypic.com/r/fy1tax/5

fy1tax.jpg


I attempted this question on MAPLE 13(shown above), but I get an obvious incorrect answer 3.17... instead of 3.14...

Am i using the formula correctly? I've double checked the values and don't think I missed an exponent or inserted the wrong sign.

-Thanks in advance.
 
Physics news on Phys.org
I'm not familiar with Maple, so I can't tell you what it's doing. However, using the same methid in Mathematica, I get 3.14... as expected, so I assume there is something odd about the way you've told Maple to calculate it...perhaps enclosing everything after "evalf" in a pair of brackets might change things?

In any case, I'm not sure you need terms all the way up to x^{51} to get an accuracy of 53 decimal places. Use the remainder theorem for Taylor expansions to figure out how many terms you actually need.
 
Question: A clock's minute hand has length 4 and its hour hand has length 3. What is the distance between the tips at the moment when it is increasing most rapidly?(Putnam Exam Question) Answer: Making assumption that both the hands moves at constant angular velocities, the answer is ## \sqrt{7} .## But don't you think this assumption is somewhat doubtful and wrong?

Similar threads

  • · Replies 9 ·
Replies
9
Views
2K
  • · Replies 30 ·
2
Replies
30
Views
3K
  • · Replies 3 ·
Replies
3
Views
2K
  • · Replies 5 ·
Replies
5
Views
1K
Replies
1
Views
1K
  • · Replies 1 ·
Replies
1
Views
3K
  • · Replies 1 ·
Replies
1
Views
2K
Replies
3
Views
2K
  • · Replies 12 ·
Replies
12
Views
10K
  • · Replies 9 ·
Replies
9
Views
2K